You are on page 1of 21

IIT JEE (M + A) ENTHUSIAST COURSE

(PHASE - 1 & 2)

INTERNAL TEST - 01 DATE: 25.05.2022

SYLLABUS

PHYSICS : Electrostatics

CHEMISTRY : Solutions

MATHS : Function (Complete)


CLASSROOM CONTACT PROGRAMME
(Academic Session : 2022 - 2023)

TARGET : JEE (M + A) : 2023


CLASS : ENTHUSIAST (XII) (PHASE - 1 & 2)
INTERNAL TEST - 01
TEST TYPE: OFFLINE DATE : 25.05.2022 PATTERN : JEE MAINS

ANSWER
PART - 1 PHYSICS
Q. No 1 2 3 4 5 6 7 8 9 10 11 12 13 14 15 16 17 18 19 20
Section - I
Ans. C B D D C D C B A C C B D B B B C A A B
Q. No 1 2 3 4 5 6 7 8 9 10
Section - II
Ans. -80 173 45 2 900 -2 3 150 6 600
PART - 2 CHEMISTRY
Q. No 1 2 3 4 5 6 7 8 9 10 11 12 13 14 15 16 17 18 19 20
Section - I
Ans. B C B B D A C C C D B D B C A A C B A C
Q. No 1 2 3 4 5 6 7 8 9 10
Section - II
Ans. 3 8 6 3 6 3 5 9 2 1
PART - 3 MATHEMATICS
Q. No 1 2 3 4 5 6 7 8 9 10 11 12 13 14 15 16 17 18 19 20
Section - I
Ans. B D D C B B C C A B A C D C A B B A A A
Q. No 1 2 3 4 5 6 7 8 9 10
Section - II
Ans. 3 28 65 14 8 9 8 7 2 1

SOLUTION
PART 1 – PHYSICS 1
2
Ex A 600 0.1 2 0.1
SECTION - I 1 0.1

1. Due to a charge inside a cube the electric field is 1


2
1/ 2 2 Ex 0.2
A 600 0.2 2 0.1
EX 600 x , Ey 0, E Z 0 . The charge inside the
cube is (approximately) 1 1

T 600 0.1 0.1 0.2 2 0.1 2

Y
0.1m
T 6 0.447 0.316
6 0.131

O q 6 0.131 0
X
12
Z q 6.95 10 C
0.1m
2. Four charges are arranged at the corners of a square
(A) 600 C (B) 60 C ABCD as shown in the figure. The force on the
(C) 7 pC (D) 6 pC charge kept at the centre O will be:
Ans. C +q +2q
A B
2
Sol. We know that
Eo
O
T 1 2

D C
-2q +q
ENTHUSIAST COURSE/PHASE-1&2/INTERNAL TEST-01/25.05.2022/SOLUTION
(A) Perpendicular to side AB q
sin 90 sin 0 ˆj
(B) Along the diagonal BD 2
2 0 R2
(C) Along the diagonal AC
(D) Zero q ˆj
E 2
Ans. B 2 0 R2
Sol. F net due to charge placed at A and C, Fnet FOA FOC O q
Hence, Electric field at point O is 2 2

0 R2
4. In a uniformly charged sphere of total charge Q and
KqQ radius R, the electric field E is plotted as a function
FOA FOC 2
a of distance from the centre. The graph which would
2 correspond to the above will be :-
E
F net due to charge placed at B and D, Fnet FOD FOB

F1 F2 towards D
(A)
If we put a positive charge at O.
R r
The resultant force due to the charge placed at A
and C is zero and resultant force due to B and D is
E
towards D, along the diagonal BD.
3. A thin semi-circular ring of radius r has a positive
charge q distributed uniformly over it. The net field
(B)
E at the centre O is:-
R r
ˆj
+ E
+ +
+
+ +
+ +
+ + (C)

O R r
q q
(A) 2 2 2
ĵ (B) 4 2
ĵ E
0 r 0 r2

q q
(C) 2 2
ĵ (D) 2
ĵ (D)
4 0 r 2 0 r2
Ans. D R r
Sol. Given, Ans. D
1 q E
k and (where, = linear charge
4 0 R
density) 1
E r E
Sol. r2
Small charge on small length Rd is, dq Rd
r
Inside r = R Outside
/2 /2
k Rd ˆj
E dE cos 2 cos
/2 0 R2

1 q R /2
ˆj
E 2 sin
4 0 R R2 0

SOLUTION 3/21
Target : JEE (M + A) 2023
5. The electric field in a region is given by
Qenclosed a2 R2 x2
E Ax B ˆi , where E is in NC 1 and x is in
metres. The values of constants are A = 20 SI unit R2 x2
and B = 10 SI unit. If the potential at x = 1 is V1 and thus,
0

that at x = -5 is V2 , then V1 V2 is :- 7. Electric charges q, q, -2q are placed at the corners


(A) -48 V (B) -520 V of an equilateral triangle ABC of side l. The
(C) 180 V (D) 320 V magnitude of electric dipole moment of the system
is
Ans. C
(A) ql (B) 2ql
Sol. E 20x 10 ˆi
(C) 3 ql (D) 4 ql
1
V1 V2 20x 10 dx Ans. C
5
Sol. Charge at A can be split into two -q charges, so we
1
V1 V2 10x 2 10x have two dipoles as shown.
5
-2q Here two dipoles are possible
V1 V2 10 25 5 1 1 -q -q
A
V1 V2 180V P P
60°
6. An infinite, uniformly charged sheet with surface
charge density cuts through a spherical Gaussian B C +q +q
surface of radius R at a distance x from its center, q q
as shown in the figure. The electric flux through Since dipole is vector quantity, so we add the two
the Gaussian surface is : dipoles vectorially to get the resultant dipole
moment.
P q
Both dipoles are at angle 60
R
X So, Resultant Pnet P2 P2 2 P P cos 60

3P 2 3P

Pnet 3 q L
8. The work done in rotating a dipole through 180° from
R2 2 R2 x2 electric field direction is
(A) (B)
0 0 (A) pE (B) 2pE

R x
2
R2 x2 pE
(C) (D) Zero
(C) (D) 2
0 0
Ans. B
Ans. D Sol. Work done in rotating a dipole
Sol. Here the infinite inside the spherical Gaussian surface
2

is a circular sheet of radius a R2 x2 W d pE cos 1 cos 2


1
Qenclosed
using Gauss's law, the electric flux, Here, 1 0 & 2 180
0

W pE cos0 pE cos180 2pE


Qin A

4/21 SOLUTION
ENTHUSIAST COURSE/PHASE-1&2/INTERNAL TEST-01/25.05.2022/SOLUTION
3/ 2
9. A particle of mass 100 gm and charge 2 C is Fr 2Kq 2 x / d 2 x2
released from a distance of 50 cm from a fixed charge
x <<< d
of 5 C . Find the speed of the particle when its
distance from the fixed charge becomes 3 m. Neglect Fr 2kq 2 x / d 3
any other force.
q2 x / 2 0 d3
(A) 3 (B) 9
= - kx
(C) 6 (D) 0.3 K q2 / 2 0 d3
Ans. A
Angular Frequency :-
Sol. Loss of potential energy = gain in kinetic energy
k/m
U1 U2 K.

1 1 q2 / 2 0 md 3
kQq 1/ 2 mv 2 0
r1 r2
11. Charges Q1 and Q 2 are at points A and B of a right
angle triangle OAB (see figure). The resultant electric
2kQq r2 r1 field at point O is perpendicular to the hypotenuse,
v
m r1 r2 then Q1 / Q 2 is proportional to:
A
2 9 109 5 10 6 2 10 6
2.5 Q1
3 m/s
0.1 3 0.5
10. Two electrons each are fixed at a distance '2d'. A x1
third charge proton placed at the midpoint is displaced
slightly by a distance x (x << d) perpendicular to the Q
line joining the two fixed charges. Proton will execute O x2 B 2
simple harmonic motion having angular frequency : x12 x13
(m = mass of charged particle) [where q = charge (A) (B)
on proton or electron] x12 x 32

1 1 x1 x2
2q 2 2
0 md
3 2
(C) x (D) x
(A) 3 (B) 2 1
0 md 2q 2
Ans. C
1 1
q2 2 2 0 md 3 2
A Q1
(C) (D)
2 0 md 3 q 2

Ans. C
x1
F1 cos P KQ2
F1cos X 22
F1 F1 90°- 0 Q2
Sol. x2 B
x
2F1sin KQ1
Sol.
X 12

Fix Fix
e d d e tan kQ2 / x 22 / kQ1 / x12 x1 / x 2
Restoring force on proton:-
Q 2 .x12 / Q1 .x 22 X1 / X 2
2 2 2
Fr 2F1 sin where F1 kq / d x
Q1 / Q 2 x1 / x 2
SOLUTION 5/21
Target : JEE (M + A) 2023
12. A cube of side 'a' has point charges +Q located at
A
each of its vertices except at the origin where the -q
charge is -Q. The electric field at the centre of cube a
is:
F
O A Q
x
x C
Sol. F
+Q +Q a 2a

+Q -q
+Q R
+Q z
+Q
-Q 1 qQ x
+Q Fnet 2Fcos 2 2 2 1/ 2
a 4 0 a x a 2
x2
y

Q i.e., Fnet 1 2qQx


(A) xˆ yˆ zˆ 4 2 3/ 2
3 3 2 0 a x2
0a

2Q As the restoring force Fnet is not linear, motion will


(B) 2
xˆ yˆ zˆ be oscillatory (with amplitude 2a) but not simple
3 3 0 a
harmonic
2Q 14. Three charges Q, +q and +q are placed at the vertices
(C) xˆ yˆ zˆ of a right angle triangle (isosceles triangle) as shown.
3 3 0 a2
The net electrostatic energy of the configuration is
Q zero, if Q is equal to:
(D) 2
xˆ yˆ zˆ
3 3 0a Q
Ans. B
Sol. We can replace - Q change at origin by +Q and -2Q.
Now due to +Q charge at every corner of the cube,
the electric field at the centre of the cube is zero so
now the net electric fields centre is only due to -2Q
+q +q
charge at origin. a
a
1 2Q xˆ yˆ zˆ q 2q
kqr 2 (A) (B)
E 1 2 2 2
r3 a
3

4 0 3 (C) -2q (D) +q


2
Ans. B
2Q xˆ yˆ zˆ
E BQ
3 3 a2 0

13. Two equal negative charges –q are fixed at point


(0, –a) and (0, a) on y-axis. A positive charge Q is a 2a
released from rest at the point (2a, 0) on the x-axis. Sol.
The charge Q will :
+q +q
(A) Execute simple harmonic motion about the origin A a C
(B) Move to the origin and remain at rest
(C) Move to infinity Net electrostatic energy of the configuration:
(D) Execute oscillatory but not simple harmonic 1 Q1Q2 1 Q 2 Q3 1 Q3Q1
U
motion 4 r1 4 r2 4 r3
0 0 0
Ans. D

6/21 SOLUTION
ENTHUSIAST COURSE/PHASE-1&2/INTERNAL TEST-01/25.05.2022/SOLUTION

1 Qq q2 Qq 16. An electric charge 10 3 C is placed at the origin


U .....(1) (0, 0) of X-Y coordinate system. Two points A and
4 0 a a 2a
B are situated at 2, 2 and (2, 0) respectively..
Put net electrostatic energy equals to zero i.e U = 0
The potential difference between the points A and B
Qq Qq q2 will be-
0
a 2a a (A) 9 V (B) Zero
(C) 2 V (D) 4.5 V
Q
Q q 0 Ans. B
2
Sol. The distance of point A from origin is
2Q Q 2q 0 2 2
rA 2 0 2 0 4 2
2q
Q The distance of point B from origin is
2 1
2 2
15. The flat base of a hemisphere of radius a with no rB 2 0 0 0 4 2
charge inside it lies in a horizontal plane. A uniform 3
Potential at A due to charge Q 10 C is
electric field E is applied at an angle with the A Q
4 VA as rA 2
vertical direction. The electric flux through the curved 4 0 rA 8 0

surface of the hemisphere is:-


Q Q
and potential at B is VB as rB 2
45° 4 0 rB 8 0

E Potential difference between A and B


Q Q
VA VB 0 volt.
8 0 8 0

17. A non-conducting semicircular disc (as shown in


figure) has a uniform surface charge density . The
ratio of electric field to electric potential at the centre
a 2E of the disc will be
a2E
(A) 2 2 (B)
2
b
2 a 2E
(C) (D) a 2 E a
2 2
1 nb / a 2
Ans. B (A) b a (B)
Sol. We know that,
2
E.ds E dScos 45 1 n b/a b a
(C) (D) n b / a
b a
Now if we make a closed surface by combining the
hemisphere and a circular disc the net flux through Ans. C
the closed surface is zero. b
2K dx b
So, is through circular and Sol. E dE 2K n
in
x a x a
out is through hemisphere b
K x dx
So, V dV K b a
curued circular
x a x
1 E a2 2
curued E a2 E 2k n b/a ln b / a
2 2
V k b a b a
SOLUTION 7/21
Target : JEE (M + A) 2023
18. A simple pendulum is suspended in a lift which is By Gauss law,
going up with an acceleration of 5 m/s2 An electric
field of magnitude 5 N/C and directed vertically E 4 r2 Q 2 Aa 2 2 Ar 2
upward is also present in the lift . The charge of the Given, E is independent of r
bob is 1 µC and mass is 1 mg . Taking g = 2 and
length of the simple pendulum l m, find the time period Hence, Q 2 Aa 2 0
of the simple pendulum (in sec).
Q
(A) 2 (B) 3 This gives A
2 a2
(C) 4 (D) 1
20. In a certain region of space, the potential is given by:
Ans. A
V k 2x 2 y2 z 2 . The electric field at the point
(1,1,1) has magnitude =
Sol. T 2
g eff
(A) k 6 (B) 2k 6
6
qE 1 5 10
g ff g 5 15 (C) 2k 3 (D) 4k 3
M 1 10 6
Ans. B

Sol. V k 2x 2 y2 z2
E 5m/s2
vˆ vˆ vˆ
E i j k
x y z
2
g ff 10
E 4k ˆi k 2y ˆj k 2z kˆ
T = 2 sec
19. The region between two concentric spheres of radii E k 4iˆ 2ˆj 2kˆ x, y,z 1,1,1
'a' and 'b' respectively (see figure), has volume
A E 24k 2 6k
charge density , where A is a constant and r
r
is the distance from the centre. At the centre of the
spheres is a point charge Q. The value of A such SECTION - II
that the electric field in the region between the 1. An electric field of 20 N/C exists along the positive
spheres will be constant, is: x-axis in space. Calculate the potential difference
VB VA in volts, where points A and B have
coordinates A(0, 0) & B (4m, 2m).
Ans. -80
Q a
Sol. A = (0,0); B = (4m, 2m)
b
E 20iˆ (constant)
r rB rA 4iˆ 2ˆj
Q Q
(A) (B) 2 b2 a 2 V E. r
2 a2

2Q VB VA 20iˆ . 4iˆ 2ˆj 80V


2Q
(C) a2 b2 (D) 2
a 2. Electric flux through a surface of area 100m2 lying
Ans. A in the xy plane is (in V - m) if E ˆi 2ˆj 3kˆ
Sol. Let us find total charge enclosed in a sphere of
radius r, Ans. 173
Sol. Given E iˆ 2ˆj 3kˆ , Area, A 100kˆ
r A
Q Q 4 r 2 dr Q 2 Ar 2 2 Aa 2
a r Electric Flux = E.A

8/21 SOLUTION
ENTHUSIAST COURSE/PHASE-1&2/INTERNAL TEST-01/25.05.2022/SOLUTION
Electric Flux E0 3
q .a
ˆi 2ˆj 3kˆ . 100kˆ 100 3 173.2 1

Closest integer is 173. 5 103 6 12 12 12


0 10 8.854 10 2.21 10 ~ 2 10
2 10 2
3. A point charge 0.5 C is located in the XY plane at
5. Three charges 0.1 coulomb each are placed in the
the point of position vector r0 2iˆ 3jˆ . What is the corners of an equilateral triangle of side 1m. If the
magnitude electric field at the point of position vector energy is supplied to this system at the rate of 200
kW, how much time in seconds would be required to
r 8iˆ 5ˆj , in S.I units move one of the charges onto the midpoint of the
Ans. 45 line joining the other two?
Ans. 900
(2,3) Sol. As the potential energy of two point charges
separated by a distance, r is given by
(0.

3
5

U q1q 2 / 4 0 r , the initial and final potential


c)

Sol. energies of the system will be


2
2
( 8, -5 ) 1 q2 0.1
9
Ui 3 3 9 10 2.7 108 J
4 0 r 1
r 6iˆ 8ˆj m 1 q2 1 q2 q2
Uf 2 4.5 108 J
4 0 r/2 4 0 r 4 0r
r 10m
So work done in changing the configuration of the
kq system
Magnitude of field
r2
W Uf U i 1.8 108 J
9 109 0.5 c Now, as energy is supplied at the rate of 200 kW
2
10 the time required to do work W is given by
= 45 V/m
W 1.8 108
t 900sec
E0x P 2 105
4. The electric field in a region is given by E i.
6. Due to electric dipole, electric field at a distance r on
Find the charge contained inside a cubical volume
bounded by the surfaces x = 0, x = a, y = 0, y = a, axial position is E1 and at distance r on equatorial
z = 0 & z = a. Take E 0 5 103 N / C , l = 2 cm and position is E 2 . If E1 a E 2 . Find the value of a.
a = 1 cm. Express the answer as p 10 12 C and fill Ans. -2
the nearest integer value of p. Sol. At a distance r from the centre is given by:
Ans. 2 1 2p
Sol. The electric flux will contribute only due to surfaces E1
4 0 r3
x = 0 and x = a because the electric filed is
perpendicular to other surfaces. At a distance r from the centre is given by:

E0 0 2 1 p
Thus, x 0 .a 0 and E2
1 4 0 r3
E0 a 2 E0 3 Therefore, E1 2E 2
x a .a .a
1 1
q E0 3
Using Gauss's law, x a .a
0 1

SOLUTION 9/21
Target : JEE (M + A) 2023
7. A point charge q is placed at one corner of a cube of
q d q P
edge a. Find the ratio of the flux through the cube to Sol.
1 2 E2 E1
the flux through that face of the cube which is not in X
contact with the charge q.
q1 4 C
Ans. 3
q2 1 C
face A
d 3m
At point P the fields balanced each other
Sol. kq 2 kq1
0
x2 d x
2

Q
1 4
q 1 q 2 2
0
Qcube Qface A x 3 x
8 0 38 0
x=3m
Qcube
3 distance fram 4 C 6m
Qface A
10. Find the value of electric potential (in volts) at the
8. Find out the electric potential in S.I units at the surface centre of a hollow metallic sphere of radius 6 m and
of a uniformly charged solid sphere if the potentials net charge 0.4 C uniformly spread over it.
at a distance 5 cm and 10 cm outwards from its
surface are 100V and 75 V respectively. Ans. 600
Ans. 150 kQ 9 109 0.4 6
Sol. Vin 10
kQ R 6
Sol. Vsurface = 600
R
At a distance sum from surface,
PART 2 - CHEMISTRY
kQ
VR 5 100V SECTION - I
R 5 1. The mole fraction of oxygen in a mixture of 7g of
kQ nitrogen and 8g of oxygen is:
R 5 (i)
100 8
(A) (B) 0.5
At a distance 10 cm fram surface 15
kQ (C) 0.25 (D) 1.0
VR 10 75V Ans. B
R 10
kQ 8 1
R 10 (ii)
75 Xo2 32 4 0.5
Sol. 7 8 1 1
From equation (i) & (ii)
28 32 4 4
kQ 1500 & R = 10
2. The vapour pressure of water depends upon
1500 (A) Surface area of container
Vsurface 150V
10 (B) Volume of container
9. Two point charges 4 C & 1 C are placed at a (C) Temperature
distance of 3m. Find the distance of the neutral point (D) Amount of water
from the 4 C charge in metres. Ans. C
Ans. 6 Sol. Depends only on temperature.

10/21 SOLUTION
ENTHUSIAST COURSE/PHASE-1&2/INTERNAL TEST-01/25.05.2022/SOLUTION

3. The boiling point of C6 H 6 ,CH 3OH, C6 H 5 NH 2 and Sol. X A 0.4 XB 0.6


C 6 H5 NO 2 are 80 C,65 C,184 C and Ps XA PAo XB PBo
212 C respectively, which will show highest
Ps 0.4 300 0.6 800 600 torr
vapour pressure at room temperature
(Ps)exp < (Ps)Raoult's
(A) C6 H 6 (B) CH 3OH
(–ve) deviation solution
(C) C 6 H5 NH 2 (D) C 6 H5 NO 2 7. Which condition is not satisfied by an ideal solution
Ans. B (A) H mixing = 0
1 (B) V mixing = 0
Sol. VP
B.P
(C) S mixing = 0
CH 3 OH C 6 H 6 C 6 H5 NH 2 C6 H 5 NO2
(D) Obeyance of Raoult's law
4. At the same temperature & same partial pressure, Ans. C
hydrogen is more soluble in water than helium. Which
Sol. Entropy increases upon mixing solute and solvent.
of them will have a higher value of KH?
8. A sample of air is saturated with benzene (vapor
(A) Hydrogen
pressure = 100 mm Hg at 298 K) at 298 K, 750 mm
(B) Helium Hg pressure. If it is isothermally compressed to one
(C) Both will have same value KH third of its initial volume, the final pressure of the
(D) Can't be said system is
Ans. B (A) 2250 torr (B) 2150 torr
(C) 2050 torr (D) 1950 torr
1
Sol. Solubility Ans. C
KH
Sol. P1V1 P2 V2
KH He
KH H2
V
5. An azeotropic mixture of two liquids boil at a lower 650V P2
3
temperature than either of them when
(A) It is saturated P2 1950
(B) It does not deviate from Raoult's law Ptotal 1950 100 2050
(C) It shows negative deviation from Raoult's law 9. Pressure cooker reduces cooking time because
(D) It shows positive deviation from Raoult's law (A) The heat is more evenly distributed inside the
Ans. D cooker
Sol. Min B.P azeotrope is shown by (+ve) deviation (B) A large flame is used
solution. (C) Boiling point of water is elevated
6. Consider a binary mixture of volatile liquids. If at (D) Whole matter is converted into steam
X A 0.4, the experimental vapor pressure of Ans. C
solution is 580 then the mixture could be Sol. Conceptual
pA 300 torr, p B 800 torr 10. At 80 C, the vapour pressure of pure liquid 'A' is
(A) CHCl3 CH 3COCH 3 520 mm Hg and that of pure liquid 'B' is 1000 mm
Hg. If a mixture solution of 'A' and 'B' boils at
(B) C6 H 5 Cl C6 H 5 Br 80 C and 1 atm pressure, the amount of 'A' in the
(C) C 6 H 6 C 6 H5 CH 3 mixture is (1 atm = 760 mm Hg)
(A) 22 mol % (B) 34 mol %
(D) C 2 H5 OH H 2 O (C) 16 mol % (D) 50 mol %
Ans. A Ans. D

SOLUTION 11/21
Target : JEE (M + A) 2023

Sol. VP s
760 X A 520 X B 1000 Sol. Ps 0.4 7 KPa 0.6 12

760 520X A 1 X A 1000 Ps 2.8 7.2


Ps 10.0
760 1000 480 X A
480 X A 240 PA
yA
PS
240
XB 0.5
480 2.8
yA
11. For an ideal solution of two components A and B, 10
which of the following is true? yA 0.28
(A) H mixing 0 zero yB 0.72
(B) A–A, B–B and A–B interactions are identical 14. A graph of vapour pressure and temperature for
(C) A–B interaction is stronger than A–A and B–B three different liquids X, Y and Z is shown below:
interactions
X Y Z

Vapour Pressure
(D) H mixing 0 zero 800

(mm Hg)
500
Ans. B 400
Sol. Conceptual 200
12. Choose the correct statement with respect to the
0 293 313 333 353
vapour pressure of a liquid among the following:
Temp
(A) Increases linearly with increasing temperature
(B) Decreases non–linearly with increasing The following inferences are made:
temperature (I) X has higher intermolecular interactions
(C) Decreases linearly with increasing temperature compared to Y
(D) Increases non–linearly with increasing (II) X has lower intermolecular interactions
temperature compared to Y
Ans. D (III) Z has lower intermolecular interactions
compared to Y
Sol. VP f T The correct inference (s) is/are:
(A) (I) (B) (III)
VP 2 H 1 1
log (C) (II) (D) (I) and (III)
VP 1
2.303R T1 T2
Ans. C
13. Liquids A and B form an ideal solution in the entire
composition range. At 350 K, the vapor pressures of 1
Sol. VP int ermolecular force of attraction
pure A and pure B are 7 103 Pa and 12 103 Pa,
respectively. The composition of the vapor in X Y Z
equilibrium with a solution containing 40 mole percent
of A at this temperature is:
VP
(A) y A 0.37; y B 0.63
(B) y A 0.28; y B 0.72
0 293 313 333
(C) y A 0.76; y B 0.24 Temp

(D) y A 0.4; y B 0.6 at 313 K VP of X is more than Y.


Ans. B So X will have lower intermolecular than Y.

12/21 SOLUTION
ENTHUSIAST COURSE/PHASE-1&2/INTERNAL TEST-01/25.05.2022/SOLUTION

15. If Raoult's law is obeyed, the vapour pressure of the 18. A liquid is kept in a closed vessel. If a glass plate
solvent in a solution is directly proportional to (negligible) mass with a small hole is kept on top of
(A) Mole fraction of the solvent the liquid surface, then the vapour pressure of the
liquid in the vessel is:
(B) Mole fraction of the solute
(A) More than what would be if the glass plate were
(C) Mole fraction of the solvent and solute removed
(D) The volume of the solution (B) Same as what would be if the glass plate were
Ans. A removed
Sol. VP X solvent (C) Less than what would be if the glass plate were
removed
16. If vapour pressures of pure liquids 'A' & 'B' are 300
(D) Cannot be predicted
and 800 torr respectively at 25 C. When these two
liquids are mixed at this temperature to form a Ans. B
solution in which mole percentage of 'B' is 92, then Sol. Vapour pressure depends only on temperature.
the total vapour pressure is observed to be 0.95 atm. 19. A 100cm3 solution is prepared by dissolving 2g of
Which of the following is true for this solution.
NaOH in water. Calculate the normality of the
(A) Vmix 0 (B) H mix 0 solution.
(C) Vmix 0 (D) H mix 0 N N
(A) (B)
Ans. A 2 4
Sol. Ps Raoult
300 0.92 800 0.08 N N
(C) (D)
6 8
340 torr
Ans. A
340
0.45 atm 2
760 Sol. N M Vf 1
40 0.1
Ps 0.95 atm
exp N
0.5
Ps Ps 2
exp theoretical

(+ve) deviation 20. Mole fraction of C3 H 5 OH 3 in a solution of 36 g of


17. The solubility of N2 in water at 300 K and 500 torr water and 46 g of glycerine is
(A) 0.46 (B) 0.36
partial pressure is 0.01 g L 1. The solubility
(C) 0.20 (D) 0.40
in g L 1 at 750 torr partial pressure is: Ans. C
(A) 0.02 (B) 0.005 Sol. Mol. wt of glyxerine 36 8 48 92
(C) 0.015 (D) 0.0075
46
Ans. C n solute 0.5 mole
92
P1 S1
Sol. 36
P2 S2 n solvent 2 mole
18
500 0.01 0.5 1
750 S2 Xsolute 0.2
2.5 5
0.01 750
S2 0.015
500

SOLUTION 13/21
Target : JEE (M + A) 2023

SECTION - II 4. If liquid A and B form an ideal solution then how


many of the following options would be true?
1. At 300 K two liquids A and B forms an ideal solution.
The vapour pressure of pure A and B are 400 mm Smix 0, G mix 0, Vmix 0, H mix 0
and 600 mm respectively. If 1 mol of A and x moles Ans. 3
of B are mixed, vapour pressure of solution becomes Sol. Fact
550 mm. What is value of x.
5. How many of following gases would be less soluble
Ans. 3 (in H2O) than NH3 at same P,T assuming that the
1 x gas which is more polar than NH3 is more soluble.
Sol. 400 600 550
1 x 1 x O 2 , CO 2 , NF3 ,SO 3 , N 2 ,CH 4
400 600x 550 550x Ans. 6
50x 150, x 3 Sol. All but NF3 are non polar gases and NF3 has less
2. In a solution mole fraction of solute is 1/6, if half of than NH3 .
the molecules of solute dimerise and remaining half
6. How many of following concentration terms are
of the solute molecules dissociate into two parts, new
temperature independent? (standard symbols are
mole fraction of solvent becomes a/10. Find the value
of a. w v w
given) %
, % , % , molality, molarity,,
Ans. 8 w v v
normality ppm
1 5
Sol. X B XA Ans. 3
6 6
w
Let us assume, n A 5, n B 1 Sol. % , ppm molality (Dont have any volume terms)
w
Asso (nB)2 7. A compound X undergoes tetramerization in solution
nB 0.25 considering 50% association, calculate total moles
(0.5+0.5) Disso of solute in solution? (Take initial moles of solute
nB + n B = 1) Fill the answer after multiplying the result by 8.
1.0
"Round off to the nearest integer"
New n B 1.25 Ans. 5
0.5
5 a Sol. 4A A4
XA new
6.25 10 t=0 1 0
50 0.5
a 8 t = t 1–0.5
6.25 4
3. At 300 K, the vapour pressure of an ideal solution 0.5 2.5
containing 1 moles of A and 3 moles of B is 5.5 dm n total 0.5 0.6 8 5
of Hg. At same temperature, if one mole of B is 4 4
added to this solution, the vapour pressure of solution 8. Find the value of KH of HCl in x 10 y format using
is increased by 0.1 dm of Hg. Find VP of B in pure
below graph? Then calculate (x + y)? (in torr)
state (in dm of Hg)
Ans. 6
HCl (torr)
Pressure of

1000
1 o 3 o o o
Sol. 5.5 PA PB 22 P A 3P B
4 4
500
1 o 4 o
5.6 PA PB 28 PAo 4PBo
5 5
0.01 0.02
o
PB 6 Mole fraction of
HCl in cyclohexane
Ans. 9

14/21 SOLUTION
ENTHUSIAST COURSE/PHASE-1&2/INTERNAL TEST-01/25.05.2022/SOLUTION

PART 3 - MATHEMATICS
1000 SECTION - I
1. Two sets A and B are as under:
500
Sol. A a, b R R : a 5 1 and b 5 1
2 2
0.02 B a, b R R:4 a 6 9 b 5 36
0.01
500 Then:
Slope tan KH 5 104 torr
0.01 (A) B A
x 5, y 4, x y 9 (B) A B
(C) A B (an empty set)
w
9. Find the normality of 36.5% HCl solution. Given (D) Neither A B nor B A
w
that dsolution 0.2gm / ml. Ans. B
Ans. 2 Sol. A a 5 1 1 a 5 1
w 4 a 6; 4 b 6
Sol. % 36.5
w 2 2
a 6 b 5
Vsolution 1L 1000 mL B 1
9 4
msolution 200gm
3 a 9
msolute 73 3 b 7
n solute 2

2 (6, 7)
N M vf 1 2
1 (6, 5) (9, 5)
(3,5)
10. How much more H2O should be added to a solution
to change its concentration from 1.0 M to 0.5 M? (6, 3)
(Take Vinitial = 1 L)
Ans. 1
Sol. M1V1 M 2 V2
1 1 0.5 V2 A B
V2 2 2. If A = { x Z ; x < 10 and x is a multiple of 3 or 4},
V V2 V1 1L where Z+ is the set of positive integers, then the total
number of symmetric relations on A is:
(A) 25 (B) 220
(C) 210 (D) 215
Ans. D
Sol. A = {3, 6, 9, 4, 8}
n n 1
2
Total number of symmetric relation = 2
5 6
2 2
215

SOLUTION 15/21
Target : JEE (M + A) 2023
3. Let P be the relation defined on the set of all real (A) 898 (B) 998
numbers such that P = {(a, b) : sec2a – tan2b = 1}. (C) 991 (D) None of these
Then P is: Ans. B
(A) Reflexive and transitive but not symmetric
4
(B) Reflexive and symmetric but not transitive x 1 x
4 4 4x
(C) Symmetric and transitive but not reflexive Sol. f x x
; f 1 x 1 x
4 2 4 2 4
(D) An equivalence relation 2
4x
Ans. D
4 2
Sol. For reflexive a, a sec 2 a tan 2 a 1 f 1 x x
4 2 4 2 4x
Reflexive.
4x 2 4x 2
For symmetric of a, b R , then b, a R f x f 1 x x x x
1
4 2 2 4 4 2
sec 2 a tan 2 b 1 1 1 1996
if f x ;f 1 x 1
2 2
1 tan a sec b 1 1 1997 1997 1997

sec 2 b tan 2 a 1 1 1996


f f 1
1997 1997
then b, a R
Symmetric. 1 2 1996
Hence f f ... f
1997 1997 1997
For transitive if a, b R and if b, c R , then
1 1 1 .... 1 998
a, c R 998

a, b R sec 2 a tan 2 b 1 ....(1) 6. Let f (x) = ax2 + bx + c, where a, b, c are rational


and f : Z Z, where Z is the set of integers. Then
b, c R sec 2 b tan 2 c 1 a + b is:
(A) A negative integer
1 tan 2 b tan 2 c 1 ....(2)
(B) An integer
(1) + (2) sec 2 a tan 2 c 1 (C) Non-integer rational number
a, c R (D) None of these
Ans. B
Transitive .
Sol. f : Z Z co-domain = Z
Equivalence is correct answer..
f(0) = c is integer
4. If f : R R, which of the following rule is NOT a f(1) = a + b + c = integer
real function: a + b = integer – c
(A) y = 4 – x2 (B) y = 3x2 = integer – integer
(C) y x x (D) y = 3x2 + 5 = integer.
7. The range of the function f (x) = 7–xPx–3 is:
Ans. C
(A) {1, 2, 3, 4, 5} (B) {1, 2, 3, 4, 5, 6}
Sol. y x x (C) {1, 2, 3} (D) {1, 2, 3, 4}
if x is negative, y becomes imaginary. Ans. C

4x 7 x
0 x 3 7 x
5. Let f x . Then value of Sol. Px 3 x 3 & x 5
4x 2
x 3, 4, 5
1 2 1996
f f .... f is:
1997 1997 1997 4 3! 2!
P0 , 3P1 , 2 P2 1, , 1, 3, 2
2! 0!

16/21 SOLUTION
ENTHUSIAST COURSE/PHASE-1&2/INTERNAL TEST-01/25.05.2022/SOLUTION
8. The domain of the definition of the function
Sol. f x log x3 1 x6
1
f x 2
log10 x 3 x is:
4 x 1 x6 x3 1 x6 x3
log
(A) 1,2 2, 1 x6 x3
(B) 1,0 1,2 3,

(C) 1,0 1, 2 2, log 1 x6 x 3 odd

(D) 2, 1 1,0 2, 11. Which of the following function(s) is identical to


Ans. C g(x) x 2:

Sol. 4 x 2 0 x 2, 2 (A) f x x2 4x 4

x3 x 0 x x2 1 0 (B) f x x 2

2
x x 1 x 1 0 x 2
(C) f x
x 2
+ +
– –1 0 – 1 x2 x 2
(D) f x
x 1
x 1, 0 1, 2
Ans. A
1 2
9. The domain of the function f x Sol. (A) x 2 4x 4 x 2 x 2
2
[x] [x] 6
Therefore, identical
is:
(B) f (–2) = 0 but g (–2) = 4
(A) , 2 4, (B) , 2 4, Therefore, not identical
(C) , 2 4, (D) None of these (C) Domain of g (x) is R but domain of f (x) is
R – {2}
Ans. A
(D) Domain of f (x) is R – {1}.
2
Sol. x x 6 0 12. If the function f : R R is defined by

x 3 x 2 0 f x x (x – sin x), then which of the following


statements is TRUE?
(A) f is one-one, but NOT onto
–2 3 (B) f is onto, but NOT one-one
x 2 or x 3 (C) f is BOTH one-one and onto
(D) f is NEITHER one-one NOR onto
x , 2 4, Ans. C

10. Function f x log e x 3 1 x 6 is: Sol. f x x x sin x


f (x) is odd continous function
(A) Even
(B) Odd x x sin x ; x 0
f x
(C) Neither even nor odd x x sin x ; x 0
(D) None of these
For x 0, f (x) 2x sin x x cos x
Ans. B
x 1 cos x x sin x 0
SOLUTION 17/21
Target : JEE (M + A) 2023
For x < 0, f '(x) = –2x + sin x + x cos x 14. The graph of the function y = f (x) is as shown in the
= x (cosx – 1) – (x – sin x) > 0 figure. Then which of the following could represent
So, f (x) strictly increases in the graph of the function y f x ?
, f (x) is one-one
(–2,1) (2,1)
x f x
(–1,0) (1,0)
x f x
Therefore, Onto. (0,–1)
13. The function f : R R, defined as
y y
2
x 6x 10 1
f x is:
3x 3 x 2 (A) (B)
(A) Injective but not surjective x x
–2 0 1 2 –2 0 1 2
(B) Surjective but not injective y y
(C) Injective as well as surjective
1 1
(D) Neither injective nor surjective
(C) (D)
Ans. D x x
–2 –1 0 1 2 –2 0 1 2
3x 3 x 2 2x 6 x2 6x 10 3 2x Ans. C
Sol. f ' x 2 2
3x 3 x

Sol. y = f(x)
2 3 2
6x 18x 6x 18 2x 6x
2 3 2
3x 2x 18x 12x 30 20x
2 2
3x 3 x y f x

3x 2 14x 12
2 15. Let f (x) = x2 and g (x) = sin x for all x R. Then the
3x 3 x 2
set of all x satisfying fogogof x gogof x ,
D = 196 – 4 (–3) (–12) where (f o g) (x) = f(g(x)), is:
= 196 – 144 > 0
(A) n ,n 0,1, 2,....
3 x x
2 (B) n ,n 1, 2,....
3x 3 x 2

f ' (x) is both positive and negative. (C) 2n ,n ....., 2, 1,0,1,2,....


2
Therefore, not injective.
(D) 2n ,n ....., 2, 1,0,1,2,....
x2 6x 10 D 0
y Ans. A
3x 3 x 2 D 0
Sol. g o f = g (f(x)) = g (x2) = sinx2
always ve g o g o f = g (sinx2) = sin (sin x2)
ve
always ve f o g o g o f = f (sin (sin(x2)) = sin2 (sin x2)
Therefore, codomain not equal to range. sin 2 sin x 2 sin sin x 2
Hence, not surjective.
sin sin x 2 sin sin x 2 1 0

18/21 SOLUTION
ENTHUSIAST COURSE/PHASE-1&2/INTERNAL TEST-01/25.05.2022/SOLUTION
Ans. A
sin sin x 2 0 or sin sin x 2 1
x2 x x 1
2 Sol. y 2
sin x n 0 x 2x x 2
x2 n sin x 2 4n 1
2 yx 2y x 1
x n not possible
1 2y x 1 y
16. The fundamental period of
sin x cos x 1 2y
is: x g y
sin x cos x sin x cos x 1 y

1 2x 1
g x f x
(A) (B) 1 x
2
2 21 x 1 2x
(C) 2 (D) g' x 2
3 1 x
Ans. B
2 2x 1 2x 3
cos x sin x 1 x
2
1 x
2
Sol. f x
2 cos x sin x cos x sin x
19. If g (x) is a polynomial satisfying g(x) g(y) = g(x) +
f x g(y) + g (xy) – 2 for all real x and y and g (2) = 5
then g (3) is equal to:
(A) 10 (B) 24
Therefore, Period
2 (C) 21 (D) None of these
Ans. A
17. The function f x sin 4x cos 2x , is a periodic
Sol. g(x) g(y) = g(x) + g(y) + g (xy) – 2
function with period:
x 2, y 1 5 g 1 5 g1 5 2
(A) 2 (B)
2 g 1 2 12 1

(C) (D) g 2 5 22 1
4
Ans. B g 3 32 1 10
20. If f (x + ay, x – ay) = axy then f (x, y) is equal to:
Sol. f x sin 4x cos 2x
x2 y2 x2 y2
(A) (B)
4 2 4 4
x2 y2
LCM , (C) (D) None of these
4 2 2 2
Ans. A
2
x x d Sol. Let x + ay = X
18. Let f x 2
,x 0, 2. Then f 1 (x)
x 2x dx x – ay = Y
(wherever it is defined) is equal to: X Y X Y
x &y
1 2 2a
3
(A) (B) 2
1 x
2 1 x X Y X Y
f X, Y a xy a
2 2a
1 3
(C) 1 x 2 (D)
1 x
2 X2 Y2
4
SOLUTION 19/21
Target : JEE (M + A) 2023
SECTION – II 4. Let E 1,2,3,4 and F 1,2 . Then the number
1. Let f be a real-valued invertible function such that of onto functions from E to F is
2x 3 Ans. 14
f 5x 2, x 2. Then the value of
x 2 Sol. From E to F we can define, in all, 2 2 2 2 16
functions (2 options for each elements of E) out of
1
f 13 is which 2 are into, when all the elements of E map to
Ans. 3 either 1 or 2
No. of onto function = 16 –2 = 14.
2x 3 1 2x 3
Sol. We have f 5x 2 f 5x 2 5. If f x satisfying the relation
x 2 x 2

Let 5x 2 13, then x 3 f x f x 4 f x 2 f x 6 for all x,

2 3 3 then prove that f x is periodic and find its


1
Hence f 13 3 period.
3 2
Ans. 8
2. Suppose that f x is a function of the form
Sol. Given f x f x 4 f x 2 f x 6 ...(1)
ax 8 bx 6 cx 4 dx 2 15x 1 Replace x by x + 2
f x x 0 .
x
f x 2 f x 6 f x 4 f x 8 ...(2)
If f 5 2, then the value of f 5 is
From equations (1) and (2), we have
Ans. 28
f x f x 8
8 6 4 2
ax bx cx dx 15x 1
Sol. f x Hence, f x is periodic with period 8.
x
2
7 5 3 1 6. If f x y 1 f x f y x, y R and
ax bx cx dx 15
x
odd f 0 1, then f 2 ..............
Now f x f x 30 Ans. 9
Sol. Put x = y = 0 f 1 4
f 5 30 f 5 28
put x = 0, y = 1 f 2 9
3. If f x is a polynomial satisfying
7. The maximum integral value of x in the domain of
1 1
f x f f x f and f 3 28, then f x log10 (log 1 log 4 x 5 is :
x x 3

f 4 is equal to Ans. 8
Ans. 65 log 1 log 4 x 5 0
Sol.
3
Sol. f x xn 1
0 log 4 x 5 1
f 3 3n 1 28
1 x 5 4
3n 27 6 x 9
n 3

f 4 43 1 65.

20/21 SOLUTION
ENTHUSIAST COURSE/PHASE-1&2/INTERNAL TEST-01/25.05.2022/SOLUTION
8. Number of integers satisfying the equation 10. The period of f x x 2x 3x
2 2
x 5x x x 6x is :
n n 1
4x ...... nx x, where n N, is
Ans. 7 2
2
Sol. x 5x x x2 6x x 2 5x x x2 (where . represents greatest integer function)
Ans. 1
x2 5x x x2 Sol. f x x 2x 3x ... nx x 2x 3x ... nx

a b a b ab 0 x 2x 3x ... nx

x 2 5x x x 2 0 1 1 1
The period of f x LCM 1, , ,..., 1.
2 3 n
x x 5 .x x 1 0 5 x 1
9. The number of real solutions of the equation
log 0.5 x 2 x is

Ans. 2
Sol. Draw the graph of y log 0.5 x and y 2 x

y 2x y 2x

y 1 1 x
0

y log0.5 x
y log0.5 x

Clearly, from the graph, there are two solutions.

SOLUTION 21/21

You might also like